Political analyst: Several years ago, McFarlane, the military dictator, had Brooks, the former prime minister, arrest...

@MichaelaJ on October 18 at 01:14AM

Am I thinking along the correct lines?

Hello- Please let me know if I am thinking along the correct lines in my interpretation of this question. Given that this is a necessary assumption question, we are looking to bridge the gap as to why Brooks would have fewer supporters. I choose A and it is incorrect because the stimulus doesn't discuss the government's legitimacy. It has really nothing to do with the argument and doesn't bridge any gaps in the argument. Is that an accurate way of looking at it? D is correct because we know that there will be McFarlane supporters who believe Brooks is guilty and opponents of McFarlane who oppose anyone who joins his government, therefore, given that Brooks is joining McFarlane's government and has let's say 1/2 of McFarlane supporters who believe she is guilty and 1/2 of McFarlane's opponents who oppose anyone joining McFarlane's government, she has a majority of people who oppose her, which leads to her having few supporters in the country. Is that an acceptable way of looking at this? Thanks!

Replies
Create a free account to read and take part in forum discussions.

Already have an account? log in

Emil-Kunkin on October 19 at 10:19PM

I think we should take a step back, for a necessary assumption I think we need to treat this as something that the author must agree with, not necessarily something that bridges the gap like a sufficient assumption question would. That said, I think the right answer is both a necessary and sufficient assumption, so in this case bridging the gap would work.

Either way we should dissect the argument. The author thinks that brooks will have next to no support. The rationale is that supporters of the military dictator think she is corrupt, and that opponents of the military dictatorship will oppose her.

This doesn't actually prove that she will have very little support among the general population. It only shows that she will have very little support among people who are opponents or supporters of the dictator. This completely ignores the (likely large) slice of the population who are lukewarm on the dictator, or simply don't care. The author wrongly assumes that everyone is either a supporter or opponent.

This is what D fixes. The author has to think that most people actually care and are supporters or opponents, otherwise her argument would make no sense.

@MichaelaJ on October 20 at 08:01PM

So we are not talking about support/opposition for Brooks, we are talking about the support/opposition of McFarlane?

Emil-Kunkin on October 21 at 03:02PM

We're really talking about both. The author is using support for Mcfarelane as a way to understand support for brooks, and while this actually works, the author fails to consider there is a category of people who neither support nor oppose M.

@MichaelaJ on October 22 at 06:06PM

I am starting to see it a bit, but am still not quite understanding. If our conclusion is focusing on the number of supporters for Brooks, why do we circle back to the number of supporters McFarlane has? Is it because the two are connected in some way?

Emil-Kunkin on November 2 at 04:00PM

The author thinks they are connected: the author uses the fact that Ms supporters will not support brooks to infer something about Brooks's support. This is totally reasonable- I think it's fair to say that if 40 percent of a state support trump, at least 40 percent of that state will not support biden. The situation is similar in the passage, if the supporters one one person all think that the other person is corrupt, we can infer that the supporters of the first will not support the second.

@MichaelaJ on November 3 at 12:37AM

That makes sense now! Thank you!